Alex has $13.00 to spend on the bus.
The bus charges $0.35 for each mile
traveled plus $1.00 a trip. Alex
spends all but $0.80 of his money on
the bus. How many miles did Alex
travel?

Answers

Answer 1

Answer:

The answer is 31.42 miles

Step-by-step explanation:


Related Questions

How do you graph the vertex of a function?

Answers

To graph the vertex of a function, first identify the vertex by calculating the x-coordinate using the formula x = -b/(2a), where a and b are the coefficients of the quadratic equation.

Then, plug the x-coordinate into the equation and solve for y to get the y-coordinate of the vertex. Finally, plot the coordinates on the graph to get the vertex of the function.

Graphing the vertex of a function requires a few steps. First, the equation of the function must be written in standard form, which is ax^2 + bx + c = 0. Next, the x-coordinate of the vertex must be calculated by using the formula x = -b/(2a).

Then, the x-coordinate is plugged into the equation and solved for y to get the y-coordinate of the vertex. Finally, both coordinates are plotted on a graph to get the vertex of the function. This process can be used to quickly graph the vertex of any quadratic equation.

Learn more about Graphs here:

https://brainly.com/question/25799000

#SPJ4

Answer: d
Explanation: Hope this helps pzzzz marke brainliest

Answers

Youre answer is D, you’re welcome

Answer:

lol think you made the wrong thing.

Convert the following equation to slope-intercept form:
12x + 12y = 36

Answers

Answer:

 Slope = -2.000/2.000 = -1.000

 x-intercept = 3/1 = 3.00000

 y-intercept = 3/1 = 3.00000

Answer:

y = -x + 3

Step-by-step explanation:

1) 12x + 12y = 36

2) 12y = -12x + 36 (carry 12x to the other side making it a negative number)

3) y = -x + 3 (divide 12 from each side)

x3+y3+z3
what is the answer pls

Answers

the answer is

=3x+yx3+zx

=3x + 3y+3z

5. Oshaunda buys a car that costs $21,000. It depreciates at 8.2% per year. a. Write an equation for the value of the car. V=21,000(1-0.082) V-21,000(0.918) B. Oshaunda tries to sell the car 4 years later. What is the car worth when it is 4 years old? Hint: Use your formula for part (a), and plug in t = 4. Use GEMA to finish the math.​

Answers

Answer:

a.

\(f(t) = 21000( {.918}^{t} )\)

b.

\(f(4) = 21000( {.918}^{4}) = 14913.86\)

Urgent question please solve and only if you know it please. Most accurate gets branliest and a new follower (me).
Step one: round each number.
step 2: add the two 10s, use your Smart second grade strategies.
1) 17+23 ---> ___+___=___
2) 42+37 ---> ___+___=___
3) 9+73 ---> ___+___=___
4) 68+21 ---> ___+___=___
5) 12+81 ---> ___+___=___​

Answers

Answer:

1)40

17+23-->20+20=40

2)80

42+37=40+40=80

3)80

9+73=10+70=80

4)90

68+21=70+20=90

5)90

12+81=10+80=90

What are the 3 properties of parallel lines?.

Answers

The three properties of parallel lines are transitive property of parallel lines, symmetric properties of parallel lines and properties cut by transversal.

Properties of parallel lines when cut by a transversal

Two lines when cut by a transversal line are parallel if the corresponding angles so formed are equal. Generally, the corresponding angles are in relative positions and lie along the same side of the transversal.

Two lines cut by a transversal line are said to be parallel if the alternate interior angles so formed are equal. In general, the pairs of the alternate interior are found in the inner side but lie on the opposite sides of the transversal.

Two lines cut by a transversal line are said to be parallell if the alternate exterior angles so formed are equal. In general, the pairs of alternate exterior angles are found on the outer side but lie opposite each other.

The above mentioned are 3 properties of parallel lines.

To learn more about transversal refer here

https://brainly.com/question/24607467

#SPJ4

There were 70 enrolled students in STAT 3355 during the year 2020 . The population of adults, 18 years or older, in the United States was 258.3 million in 2020 . A student surveyed 30 of her classmates in 2020 and found that 22 students liked to play video games. If this student computed a 95% confidence interval, would it have contained the value of 65%, which was known to be the proportion of adults that liked to play video games in the United States in 2020. (Hint: Calculate the confidence interval by hand at first, and then try to use R ).

Answers

The confidence interval for proportion of adults who liked video game is (0.575091,0.8915756 ) from sample. It has a parameter 65%.

Number of enrolled students in STAT during year 2020 = 70

The population of adults that is 18 or above in 2020 = 258.3 million

Number of students are classmates= 30

Out of 30, number of students who like video game = 22

level of significance = 0.95

Let p denote the proportion of students in sample who liked video games. it is p = 22/30 = 0.733.

Using the distribution table, value of z for 95% is equals to the 1.96. From the formula of confidence interval, \(CI = p ± z\sqrt{ \frac{ p( 1 - p)}{n}}\)

\(= 0.733 ± \sqrt{ \frac{0.733( 1 - 0.733)}{30}}\)

= (0.575091, 0.8915756), the lower limit and upper limit of interval. This interval contains 65% which is population parameter. Hence, required value is (0.575091, 0.8915756).

For more information about confidence interval, visit:

https://brainly.com/question/15712887

#SPJ4

If everyone in a class scored 100 on a quiz, what is the standard deviation of quiz scores?

Answers

The standard deviation of quiz scores is 0.

What is the standard deviation?The standard deviation in statistics is a measure of the amount of variation or dispersion in a set of values. A low standard deviation indicates that the values of the set tend to be close to the mean (also known as the expected value), whereas a high standard deviation indicates that the values are spread out over a larger range.

To find the standard deviation:

Let, the number of students = nx = 100Mean = n×x/n=xVariance = \(\frac{\sum(\bar{x}-x)^2}{n}\)Variance = (x₁ - x)² + (x₂ - x)² + (x₃ - x)²/nV = 0Standard variance = √0

Therefore, the standard deviation of quiz scores is 0.

know more about standard deviation here:

https://brainly.com/question/475676

#SPJ4

A wall separates an office from a laboratory. The required sound reduction index between the two spaces is 45 dB at 1000 Hz. The wall, of total area 25 m², is built of concrete block 120 mm thick with a sound reduction index of 70 dB and a window. What is the maximum size of window (in m2), formed of glass with a sound reduction index of 27 dB, that can be used to ensure an overall sound reduction index of 45 dB at 1000 Hz? Discuss the relevance of other pathways sound might take between the two rooms

Answers

The maximum size of the window is approximately 1.84 m². To calculate it, subtract the sound reduction index of the concrete block (70 dB) from the required index (45 dB) to find the remaining reduction needed (25 dB).

Then, divide this value by the sound reduction index of the glass (27 dB) to determine the maximum window area. The concrete block provides a sound reduction index of 70 dB. Subtracting this from the required index of 45 dB leaves a remaining reduction of 25 dB. The glass window has a sound reduction index of 27 dB. Dividing the remaining reduction by the glass index (25 dB / 27 dB) yields a maximum window area of approximately 0.9259. Since the total wall area is 25 m², the maximum window size is approximately 1.84 m². To achieve a sound reduction index of 45 dB at 1000 Hz, the maximum size of the window should be approximately 1.84 m².

Other sound pathways between the office and laboratory, such as doors or ventilation systems, should also be considered to ensure effective noise control.

To know more about  concrete  visit:

https://brainly.com/question/32805749

#SPJ11

of the 50 players in a school band 12% play a woodwind instrument how many players play a woodwind instrument ​

Answers

Answer:

6

Step-by-step explanation:

multiply 50 by .12 to get 6

A company purchases a copier for $2000 and the material for each order costs $15. a) How many orders must be printed for the average cost per order to fall to $65? (Show Work) b) What happens to the average cost as more orders are printed?​

Answers

Answer:

a) 40 orders;b) Average cost decreases.

------------------------------------

a) Use the following formula:

average cost = total cost/number of orders

The total cost is the cost of the copier plus the cost of the materials for each order. So, we have:

65x = 2000 + 15x, where x is the number of orders.

We can simplify this equation to:

2000 = 50x x = 40

Hence the average cost per order is $40.

b) As more orders are printed, the average cost per order will decrease because the fixed cost of the copier is spread over a larger number of orders. In other words, the more orders that are printed, the lower the average cost per order will be.

luz drives at an average speed of 60 miles per hour.
She has driven for 2 hours and has traveled a distance of 120 miles.

This situation can be represented with a linear equation written in point-slope form, where x represents the number of hours and y represents the number of miles.

Answers

Answer:

The answer is y = 60x, there is no y-intercept as she has started at zero.

Step-by-step explanation:

Hope this helps!

Answer:

i got the first one wrong(not 4)

the second is (2,120)

and the last y-120=60(x-2)

Step-by-step explanation:

Which figure is translation figure p

Answers

Answer:

Figure M

Step-by-step explanation:

Figure P has vertical at points (-2,4),(-2,8),(-6,8) and (-6,6)

Consider Figure M with verticles (-4,-4),(-4,-8),(-8,-8) and (-8,-6).

1.Translate Figure M 2 units to the right this translation has the rule.

(x,y)--(x,2y),

So

: (-4-4)--(-2,-4);

: (-4,-8)--(-2,-8);

: (-8,-8)--(-6,-8);

: (-8,-6)--(6-6);

2. Reflect the translator image Figure M across the Z-axis according to the rule.

(x,y)--(x,y)

thus,

:(-2,-4)--(-2,4);

: (-2,-8)--(-2,8);

: (-6,-8)--(-6,8);

: (-6,-6)--(-6,6);

CORRECT ME IF IM WRONG PA BRAINLIEST PO.

Let R be the region in the first quadrant bounded by the x-axis, the graph of x=y2+2, and the line x=4. What is a the interval for the area of R?

Answers

A region in the first quadrant defined by the x-axis, the graph of x=y2+2, and the line x=4 is called R then the area of R be \($-\frac{2 \sqrt{2}}{3}+2 \sqrt{2}$$\)

What is meant by parabola ?

A parabola is a U-shaped plane curve in which every point is situated at an equal distance from both the focus, a fixed point, and the directrix, a fixed line. A parabola is a U-shaped curve that represents the graph of a quadratic function.

Students are able to fill in the gap between the mathematics they learn in class and the things they see in real life by drawing a comparison between the equation of a parabola and a shape found in the real world (the parabaloid). They can conceive difficult ideas that they might otherwise find repugnant by doing this.

Sketch the graph of \($x=y^2+2$\) by sketching the sideways parabola \($x=y^2$\) then adding 2 to the x coordinates (translate 2 to the right).

Add the x axis and the vertical line x = 4.

The area can be found by either

\($\int_2^4 \sqrt{x-2} d x$$\) or \($\int_0^{\sqrt{2}}\left(4-\left(y^2+2\right)\right) d x$$\)

\($=\int_0^{\sqrt{2}}-x^2+2 d x$$\)

Apply the Sum Rule: \($\int f(x) \pm g(x) d x=\int f(x) d x \pm \int g(x) d x$\)

\($=-\int_0^{\sqrt{2}} x^2 d x+\int_0^{\sqrt{2}} 2 d x$$\)

simplifying the equation, we get

\($\int_0^{\sqrt{2}} x^2 d x=\frac{2 \sqrt{2}}{3}$$\)

\($\int_0^{\sqrt{2}} 2 d x=2 \sqrt{2}$$\)

\($=-\frac{2 \sqrt{2}}{3}+2 \sqrt{2}$$\)

To learn more about parabola refer to:

https://brainly.com/question/2841375

#SPJ4

Let R be the region in the first quadrant bounded by the x-axis, the graph of x=y2+2, and the line x=4.

8z + 6 = 7z + 4
I need help with it please show the steps

Answers

z=2 thats the answer to your problem

Find the greatest common factor of 20 and 25. Factors of 201 Factors of 25: ____ Greatest common factor: ____ please help me​

Answers

Answer:

factors of 20: 1, 2, 4 , 5, 10, 20

factors of 25: 1, 5, 25

gcf: 5

Step-by-step explanation: the common factors of 20 and 25 is 1 and 5. the greater one is 5, therefore the gcf is 5.

hope this helps!! :)

Consider a population of wildflowers in which the frequency of the red allele cr is p = 0.7.

Answers

The wildflowers in this population, the white allele is present in about 30% of the individuals, while the red allele is present in about 70% of the individuals.

The frequency of the white allele (CW) in the population can be determined by subtracting the frequency of the red allele (CR) from 1, since the frequencies of all alleles in a population must add up to 1.

So, the frequency of the white allele (CW) can be calculated as follows:

CW = 1 - CR

Given that the frequency of the red allele (CR) is p = 0.7, we can substitute this value into the equation:

CW = 1 - 0.7

  = 0.3

Therefore, the frequency of the white allele (CW) in this population is 0.3.

In genetic terms, alleles are alternative forms of a gene, and the frequencies of different alleles within a population can be used to study genetic variations. In this case, we are considering a population of wildflowers and examining the frequencies of the red allele (CR) and white allele (CW).

The total frequency of alleles in a population is always 1 since each individual carries two alleles (one from each parent). Therefore, the frequency of the white allele can be obtained by subtracting the frequency of the red allele (0.7 or 70%) from 1. This is because the sum of the frequencies of all alleles must equal 1.

By performing the calculation, we find that the frequency of the white allele in this population is 0.3 or 30%.

This means that among the wildflowers in this population, the white allele is present in about 30% of the individuals, while the red allele is present in about 70% of the individuals.

To know more about Frequency here

https://brainly.com/question/29739263

#SPJ4

Complete Question

Consider a population of wildflowers in which the frequency of the red allele CR is p = 0.7.

What is the frequency of the white allele (CW ) in this population?

Determine how many TRIANGLES can be constructed with sides measuring 4 cm, 6 cm, and 9 cm.

a) State whether there are NO TRIANGLES, MORE THAN 1 TRIANGLE, OR 1 TRIANGLE.

b) Show your work to prove your answer from part a.

Answers

Answer:   Therefore, the answer is:

a) ONE TRIANGLE

b) The sides 4 cm, 6 cm, and 9 cm can form a single triangle.

Step-by-step explanation:

a) There is exactly ONE TRIANGLE that can be constructed with sides measuring 4 cm, 6 cm, and 9 cm.

b) To determine the number of triangles that can be formed from three given sides, we can use the triangle inequality theorem which states that the sum of any two sides of a triangle must be greater than the third side.

Let's check if the given sides satisfy this inequality:

4 cm + 6 cm > 9 cm (True)

4 cm + 9 cm > 6 cm (True)

6 cm + 9 cm > 4 cm (True)

Since all three inequalities are true, we can conclude that a triangle can be formed using these three sides.

Now, let's determine if there is only one possible triangle that can be formed or if there are more than one. To do this, we can use the fact that the largest side of a triangle must be smaller than the sum of the other two sides. If this is not the case, then it's not possible to form a triangle.

In this case, the largest side is 9 cm. Let's check if it's smaller than the sum of the other two sides:

4 cm + 6 cm = 10 cm (True)

Since 9 cm is smaller than the sum of the other two sides, we can conclude that it's possible to form a triangle, but there is only one possible triangle that can be formed.

Find the derivative of
y = arcsin(x)​

Answers

Answer:

f'(x)= \(\frac{1}{\sqrt{1-x^{2} } }\)

Step-by-step explanation:

As khown the derivative of arcsin(x) is;

f'(x)= \(\frac{1}{\sqrt{1-x^{2} } }\)

Answer:

1 /√( 1 - x^2).

Step-by-step explanation:

y = arcsin x

x = sin y

dx/dy = cos y

dy/dx = 1 / cos y

Now cos y = √( 1 - sin^2 y)

but sin y = x so

cos y = √( 1 - x^2).

So dy/dx = 1 / √( 1 - x^2).

4(x3 + xy2) dV, where E is the solid in the first octant that lies beneath the paraboloid z = 1 − x2 − y2.

Answers

To calculate the given volume integral, 4(x^3 + xy^2) dV, over the solid E in the first octant beneath the paraboloid z = 1 - x^2 - y^2, we need to set up the integral in cylindrical coordinates. The integral will involve integrating over the appropriate limits and applying the volume element in cylindrical coordinates.

In cylindrical coordinates, we have x = r cos θ, y = r sin θ, and z = z.
The equation of the paraboloid, z = 1 - x^2 - y^2, can be expressed as z = 1 - r^2.
The given volume integral becomes 4(x^3 + xy^2) dV = 4(r^3 cos^3 θ + r^3 cos θ sin^2 θ) r dz dr dθ.
To determine the limits of integration, we need to consider the region of the solid E in the first octant. Since the solid lies beneath the paraboloid z = 1 - x^2 - y^2, the upper limit for z is given by z = 1 - r^2.
The limits for r and θ depend on the region in the first octant. We need to set appropriate limits to cover the desired region.Once we have the limits for r, θ, and z, we can set up the triple integral using the volume element in cylindrical coordinates.
By evaluating the integral with the corresponding limits, we can find the value of the given volume integral over the solid E in the first octant beneath the paraboloid.

learn more about Integral here

https://brainly.com/question/31109342



#SPJ11

In the figure shown, suppose m angle ABC=n and m angle ABD=2(m angle DBC). The angle bisector of angle DBC is BE. What is m angle EBC

Answers

Answer: all I know is that the answer is n/6 cause I kept getting it wrong and it told me.

Which of the following statements are true regarding the margin of error? Select all correct options. A large confidence level requires a small value of 2*, Being more confident increasing confidence level) will yield a larger margin of error The margin of error is not influenced by the sampling distribution's standard deviation Selecting a larger SRS from the population will yield a smaller margin of error.

Answers

The following statements are true regarding the margin of error: Being more confident (increasing confidence level) will yield a larger margin of error and selecting a larger SRS from the population will yield a smaller margin of error.

A margin of error (MOE) is the degree of inaccuracy in estimating a population's true proportion or mean by analyzing a sample dataset. It represents the uncertainty or confidence level of a poll's findings. It is a critical element in political polls, scientific studies, and marketing research. The accuracy of the findings is expressed in the margin of error, which is given as a percentage. The standard deviation of the sampling distribution, not the standard deviation of the population, determines the margin of error. As a result, selecting a larger sample size, the standard deviation decreases, and the margin of error decreases. Therefore, selecting a larger SRS from the population will yield a smaller margin of error. Confidence intervals are used to determine the margin of error, and increasing the confidence level will result in a larger margin of error. As a result, being more confident (increasing confidence level) will yield a larger margin of error.

Thus, the correct options are (B) and (D). Option (A) is incorrect because a large confidence level requires a large value of 2*. Option (C) is incorrect because the margin of error is influenced by the standard deviation of the sampling distribution.

Learn more about margin of error visit:

brainly.com/question/29419047

#SPJ11

What is the standard form of y=4/9x-3

Answers

Answer: 4x-9y=27 is this equation in standard form.

Brian drives a distance of 250 m at a speed of 50 m/sec. With what speed should he
drive so that he covers twice the distance in the same time?
2 Marks​

Answers

Answer:

100 m/s

Step-by-step explanation:

This can be done without a formula

If you think about it, as you walk faster you cover more distance (taking the same duration)

So if you double your velocity, you will double the distance you cover

Ps. Idk if this still helpful,

Have a great day /night

If a tank is filling up with water from the river at a rate of 1 foot per day and 1 foot of water is approximately 300 gallons, write and equation in terms of gallons and in terms of feet?

How many gallons will there be after 16 days? how mant feet of the tank is full?

Answers

Answer:

4800 Galons, and 16 feet of water.

- Brainliest if this was helpful please :)

Please answer all 4 questions. Thanks in advance.
1. What is the present value of a security that will pay $14,000 in 20 years if securities of equal risk pay 3% annually? Do not round intermediate calculations. Round your answer to the nearest cent.
2. Your parents will retire in 19 years. They currently have $260,000 saved, and they think they will need $1,300,000 at retirement. What annual interest rate must they earn to reach their goal, assuming they don't save any additional funds? Round your answer to two decimal places.
3. An investment will pay $150 at the end of each of the next 3 years, $250 at the end of Year 4, $350 at the end of Year 5, and $500 at the end of Year If other investments of equal risk earn 12% annually, what is its present value? Its future value? Do not round intermediate calculations. Round your answers to the nearest cent. What is the present value? What is the future value?
4. You have saved $5,000 for a down payment on a new car. The largest monthly payment you can afford is $300. The loan will have a 9% APR based on end-of-month payments. What is the most expensive car you can afford if you finance it for 48 months? What is the most expensive car you can afford if you finance it for 60 months? Round to nearest cent for both.

Answers

1. The present value of the security is approximately $7,224.45.

2. The annual interest rate they must earn is approximately 14.75%.

3. The present value of the investment is approximately $825.05 and the future value is approximately $1,319.41.

4. The most expensive car they can afford if financed for 48 months is approximately $21,875.88 and if financed for 60 months is approximately $25,951.46.

1. To calculate the present value of a security that will pay $14,000 in 20 years with an annual interest rate of 3%, we can use the formula for present value:

Present Value = \(\[\frac{{\text{{Future Value}}}}{{(1 + \text{{Interest Rate}})^{\text{{Number of Periods}}}}}\]\)

Present Value = \(\[\frac{\$14,000}{{(1 + 0.03)^{20}}} = \$7,224.45\]\)

Therefore, the present value of the security is approximately $7,224.45.

2. To determine the annual interest rate your parents must earn to reach a retirement goal of $1,300,000 in 19 years, we can use the formula for compound interest:

Future Value =\(\[\text{{Present Value}} \times (1 + \text{{Interest Rate}})^{\text{{Number of Periods}}}\]\)

$1,300,000 = \(\[\$260,000 \times (1 + \text{{Interest Rate}})^{19}\]\)

\(\[(1 + \text{{Interest Rate}})^{19} = \frac{\$1,300,000}{\$260,000}\]\)

\(\[(1 + \text{{Interest Rate}})^{19} = 5\]\)

Taking the 19th root of both sides:

\(\[1 + \text{{Interest Rate}} = 5^{\frac{1}{19}}\]\\\\\[\text{{Interest Rate}} = 5^{\frac{1}{19}} - 1\]\)

Interest Rate ≈ 0.1475

Therefore, your parents must earn an annual interest rate of approximately 14.75% to reach their retirement goal.

3. To calculate the present value and future value of the investment with different cash flows and a 12% annual interest rate, we can use the present value and future value formulas:

Present Value = \(\[\frac{{\text{{Cash Flow}}_1}}{{(1 + \text{{Interest Rate}})^1}} + \frac{{\text{{Cash Flow}}_2}}{{(1 + \text{{Interest Rate}})^2}} + \ldots + \frac{{\text{{Cash Flow}}_N}}{{(1 + \text{{Interest Rate}})^N}}\]\)

Future Value = \(\text{{Cash Flow}}_1 \times (1 + \text{{Interest Rate}})^N + \text{{Cash Flow}}_2 \times (1 + \text{{Interest Rate}})^{N-1} + \ldots + \text{{Cash Flow}}_N \times (1 + \text{{Interest Rate}})^1\)

Using the given cash flows and interest rate:

Present Value = \(\[\frac{{150}}{{(1 + 0.12)^1}} + \frac{{150}}{{(1 + 0.12)^2}} + \frac{{150}}{{(1 + 0.12)^3}} + \frac{{250}}{{(1 + 0.12)^4}} + \frac{{350}}{{(1 + 0.12)^5}} + \frac{{500}}{{(1 + 0.12)^6}} \approx 825.05\]\)

Future Value = \(\[\$150 \times (1 + 0.12)^3 + \$250 \times (1 + 0.12)^2 + \$350 \times (1 + 0.12)^1 + \$500 \approx \$1,319.41\]\)

Therefore, the present value of the investment is approximately $825.05, and the future value is approximately $1,319.41.

4. To determine the maximum car price that can be afforded with a $5,000 down payment and monthly payments of $300, we need to consider the loan amount, interest rate, and loan term.

For a 48-month loan:

Loan Amount = $5,000 + ($300 \(\times\) 48) = $5,000 + $14,400 = $19,400

Using an APR of 9% and end-of-month payments, we can calculate the maximum car price using a loan calculator or financial formula. Assuming an ordinary annuity, the maximum car price is approximately $21,875.88.

For a 60-month loan:

Loan Amount = $5,000 + ($300 \(\times\) 60) = $5,000 + $18,000 = $23,000

Using the same APR of 9% and end-of-month payments, the maximum car price is approximately $25,951.46.

Therefore, with a 48-month loan, the most expensive car that can be afforded is approximately $21,875.88, and with a 60-month loan, the most expensive car that can be afforded is approximately $25,951.46.

For more questions on annual interest rate:

https://brainly.com/question/31261623

#SPJ8

in a random sample of 400 registered voters, 120 indicated they plan to vote for trump for president. determine a 95% confidence interval for the proportion of all the registered voters who will vote for trump. a. (0.25, 0.34) b. (0.29, 0.30) c. (0.27, 0.32) d. cannot be determined from the information given.

Answers

A 95% confidence interval for the proportion of all the registered voters who will vote for trump is  (0.25, 0.34).

What is a random sample?

A simple random sample is a subset of people chosen at random from a larger group, all of whom were chosen with the same probability. It is a method of choosing a sample at random.

Here, we have

Given, n = 400

x = 120

95% of confidence interval = z = 1.96

p = 120/400 = 0.3

Now by applying the confidence interval formula, we get

= 0.3 ± 1.96(√{0.3(1-0.3)}/400)

= 0.3 ± 0.0449

= (0.25, 0.34)

Hence,  a 95% confidence interval for the proportion of all the registered voters who will vote for trump is  (0.25, 0.34).

To learn more about the confidence interval from the given link

https://brainly.com/question/17097944

#SPJ4

Assume the utility function, U(X,Y)=100XY+X+2Y, where X and Y are consumption goods. Furthermore, assume that 1000kr has to be spent by the individual, and that the unit price of X is 2kr and the price of Y is 4kr. a) Find the values of X and Y that maximizes the utility. Use the substitution method. Show your calculations and assume that the second order conditions are satisfied. b) Find the values of X and Y that maximizes the utility. Use the Lagrangean method. Show your calculations and assume that the second order conditions are satisfied.

Answers

a) The values of X and Y that maximize the utility are X = 250 and Y = 125.

b) T he values of X and Y that maximize the utility using the Lagrangean method are X = 250 and Y = 125.

a) To find the values of X and Y that maximize the utility using the substitution method, we can start by substituting the budget constraint into the utility function.

Since the budget constraint is given by 2X + 4Y = 1000 (kr), we can rearrange it to solve for X: X = (1000 - 4Y)/2 = 500 - 2Y.

Now substitute this expression for X in the utility function: U(Y) = 100(500 - 2Y)Y + (500 - 2Y) + 2Y.

Expand and simplify the expression: U(Y) = 50000Y - 200Y^2 + 500 - 2Y + 2Y.

Combine like terms: U(Y) = -200Y^2 + 50000Y + 500.

To maximize the utility, we need to find the critical points. Take the derivative of U(Y) with respect to Y and set it equal to zero:

dU(Y)/dY = -400Y + 50000 = 0.

Solve for Y: Y = 50000/400 = 125.

Now substitute this value of Y back into the budget constraint to find the corresponding value of X:

2X + 4(125) = 1000,
2X + 500 = 1000,
2X = 500,
X = 250.



b) To find the values of X and Y that maximize the utility using the Lagrangean method, we need to set up the following equation:

L(X,Y,λ) = U(X,Y) - λ(2X + 4Y - 1000),

where λ is the Lagrange multiplier.

Taking the partial derivatives of L with respect to X, Y, and λ, we have:

dL/dX = 100Y + 1 - 2λ,
dL/dY = 100X + 2 - 4λ,
dL/dλ = -(2X + 4Y - 1000).

Setting these derivatives equal to zero, we have the following system of equations:

100Y + 1 - 2λ = 0,
100X + 2 - 4λ = 0,
2X + 4Y - 1000 = 0.

Solving this system of equations, we find:

λ = 1/2,
X = 250,
Y = 125.

Know more about the Lagrangean method,

https://brainly.com/question/4609414

#SPJ11

2/3 as afraction with a denominator of 6

Answers

Answer:

4/6

Step-by-step explanation:

3 x 2 = 6

2 x 2 = 4

2/3 = 4/6

Other Questions
Drag numbers into the blanks to rewrite the square roots. square root -5 (it doesnt let me put the symbol. whats the answer? The function f(x) is graphed below. what is true about the graph on the interval from x = y to x = ?* it is positive and increasing* it is positive and decreasing * it is negative and increasing* it is negative and decreasing Refer to Exhibit 1. The profit-maximizing single-price monopolist's maximum profit is simplify; 2 4/5+ 6 2/3A. 8 7/15B. 8 3/4C 8 22/15D 9 7/15 HELP HELP HELP!! Giving brainliestOf 200 students, 150 like Math or English. 50 like both subjects and 90 of the students like math. How many students like English? pls i need help on this quick pls how many btus were consumed in the united states in 2017 by consumers that used petroleum as a fuel source? responses Which Scandinavian city is known for its saunas The u. S. Policy of making military, political, and ideological commitments to countries vulnerable to the influence of communism was known as ______. Are arranged marriages more successful?. ?Half of the sum of a number n and 17 is equal to 6. What is the number n? state three reasons why air is Chassifield as a mixtureOne is through sublimation After common ragweed etched Japan a century ago and escaped their leaf beetle predators, populations of the plants evolved reduced investment in defense and greater investment in growth. During the past 30 years ragweed leaf beetles have appeared in some parts of mainland Japan. Which of the following scenarios is most likely now happening as a result?A. Japanese ragweed is evolving greater investment in defense and greater investment in growth bc both defense and growth are beneficialB. Japanese ragweed is evolving back toward greater investments in defense and reduced investment in growth as a result of selection by herbivore from beetlesC. Japanese ragweed is not evolving a new investment strategy bc it takes more than a couple of decades for a popular to evolveD. Japanese ragweed is not evolving a new investment strategy bc the beetles no longer eat the plants Margaret wants to go for a swim, and decides to jump in using the diving board that measures 3-m long.She initially leaps with a velocity of 8.0 m/s at an angle of 80 to the horizontal. What are the horizontal and vertical components of her velocity?please help if you can! How do you draw a linear graph? The function used in excel to find the "line of best fit" is trendline dataline sales residual plot what is 1/4 divided by 3/8 this is another one ......... Andrea was asked to find the value of 32 9 5/4 . How can Andrea make this problem easier to compute? A produce store sold 63 red apples. If the ratio of red apples to green apples sold was 7: 2, what is the combined amount of red and green apples sold?